Difference between revisions of "2007 AMC 12A Problems/Problem 3"

(wik)
 
(3 intermediate revisions by 3 users not shown)
Line 3: Line 3:
  
 
<math>\mathrm{(A)}\ 4\qquad \mathrm{(B)}\ 8\qquad \mathrm{(C)}\ 12\qquad \mathrm{(D)}\ 16\qquad \mathrm{(E)}\ 20</math>
 
<math>\mathrm{(A)}\ 4\qquad \mathrm{(B)}\ 8\qquad \mathrm{(C)}\ 12\qquad \mathrm{(D)}\ 16\qquad \mathrm{(E)}\ 20</math>
== Solution ==
 
'''Solution 1'''
 
Let <math>n</math> be the middle term. Then <math>n+1=3(n-1) \Longrightarrow 2n = 4 \Longrightarrow n=2</math>
 
*Thus, the answer is <math>(2-1)+(2+1)=4 \mathrm{(A)}</math>
 
  
----
+
==Solution 1==
'''Solution 2'''
+
Let <math>n</math> be the smaller term. Then <math>n+2=3n \Longrightarrow 2n = 2 \Longrightarrow n=1</math>
 +
*Thus, the answer is <math>1+(1+2)=4 \mathrm{(A)}</math>
 +
 
 +
 
 +
==Solution 2==
 
* By trial and error, 1 and 3 work. 1+3=4.
 
* By trial and error, 1 and 3 work. 1+3=4.
  
Line 16: Line 16:
  
 
[[Category:Introductory Algebra Problems]]
 
[[Category:Introductory Algebra Problems]]
 +
{{MAA Notice}}

Latest revision as of 02:09, 16 February 2021

Problem

The larger of two consecutive odd integers is three times the smaller. What is their sum?

$\mathrm{(A)}\ 4\qquad \mathrm{(B)}\ 8\qquad \mathrm{(C)}\ 12\qquad \mathrm{(D)}\ 16\qquad \mathrm{(E)}\ 20$

Solution 1

Let $n$ be the smaller term. Then $n+2=3n \Longrightarrow 2n = 2 \Longrightarrow n=1$

  • Thus, the answer is $1+(1+2)=4 \mathrm{(A)}$


Solution 2

  • By trial and error, 1 and 3 work. 1+3=4.

See also

2007 AMC 12A (ProblemsAnswer KeyResources)
Preceded by
Problem 2
Followed by
Problem 4
1 2 3 4 5 6 7 8 9 10 11 12 13 14 15 16 17 18 19 20 21 22 23 24 25
All AMC 12 Problems and Solutions

The problems on this page are copyrighted by the Mathematical Association of America's American Mathematics Competitions. AMC logo.png